Suppose that one factory inputs its goods from two different plants, A and B, with different costs, 4 and 6 each respective. And suppose the price function in the market is decided as p(x,y)= 100−x−y where x and y are the demand functions and 0≤x,y. Then as
x=
y=
the factory can attains the maximum profit,

Answers

Answer 1

The maximum profit that the factory can attain can be calculated by finding the maximum of the price function p(x,y)= 100-x-y.

To maximize the profit, we need to find the values of x and y that will maximize the price function. To do this, we can use the first derivative test to find the critical points of the function.

Differentiate the price function with respect to x and y, we obtain

p'(x) = -1

p'(y) = -1

Setting both derivatives to zero and solving for x and y, we obtain x= y= 100

To check if this is a maximum, we use the second derivative test.

Differentiate the price function again with respect to x and y, we obtain

p''(x) = 0

p''(y) = 0

Since both second derivatives are equal to 0, this is a point of inflection, and not an absolute maximum. Therefore, we need to check the boundary.

Since x and y must be positive, the boundary would be x=0 and y=0.

Substituting these values into the price function, we obtain

p(0,0) = 100

Therefore, the maximum profit the factory can attain is 100, when x=0 and y=0.

Learn more about profit at : https://brainly.com/question/29585616

#SPJ4


Related Questions

8.) Graph f(x)=-x-3
a.) Evaluate f(-4).
b.) Evaluate f(5).
c.) Evaluate f(0).
d.) Circle any ordered pairs that are included in the function:
(-3,0) (-10,7) (13,10)

Answers

From the the graph f(x) = -x-3

Part a

The value of f(-4) = 1

Part b

The value of f(5) = -8

Part c

The value of f(0) = -3

The function is given that

f(x) = -x-3

The function is the mathematical statement that shows the relationship between the independent variable and dependent variable. The function consist of the different variables, numbers and mathematical operator.

The function is

f(x) = -x - 3

Part a

The value of f(-4)

Substitute the value of x in the function

f(-4) = - (-4) - 3

= 4 - 3

= 1

Part b

The value of f(5)

f(5) = -5 - 3

= -8

Part c

The value of f(0)

f(0) = -0 - 3

= -3

Therefore, the value of f(-4), f(5) and f(0) are 1, -8 and -3 respectively

Learn more about function here

brainly.com/question/15165011

#SPJ4

Suppose that you were trying to use the Putzer method to exponentiate the matrix A which has eigenvalues dı = 4 and A2 = 6, in that order. ao(t) is always the function et. Choose the correct equation for a1(t) da1 О (а) 6a1 + e4t a1(0) = 1 dt da1 : 4a1 + e4t P (g) da1 a1(0) = 1 (c) dt -4a1 + ett a1(0) = 0 da1 (d) dt 4a1 + et a1(0) = 0 da1 (e) : 6a1 + e4t a1(0) = 0 dt da1 (f) dt — 4а, + e# a,(0) — 1

Answers

The correct equation for a1(t) is dt -4a1 + e4t a1(0) = 0.

What is equation?

An equation is a mathematical expression that shows the relationship between two values or variables. It typically consists of an equal sign (=), two expressions separated by an operator, and sometimes additional symbols like parentheses, fractions, or exponents. Equations are used to describe physical laws and conditions, solve problems, and make predictions. For example, the equation for the area of a rectangle is A = lw, where A is the area, l is the length, and w is the width.

This is because the eigenvalue of A which corresponds to the coefficient of a1(t) is 4, so the equation should contain e4t. The initial condition is a1(0) = 0, since the initial value of a1(t) is 0.

To learn more about equation
https://brainly.com/question/22688504
#SPJ4

a survey of 800 randomly selected adults in a certain country found that 82% believed that protecting the right of those with unpopular views is a very important component of a strong democracy. verify the central limit theorem conditions

Answers

On solving the question - At α = 0.05, two tailed critical value, Lower Bound =[tex]0.757[/tex], Upper Bound = 0.803, (0.757, 0.803)

What is 95% Confidence interval?

An estimation range for an unknowable parameter is referred to as a confidence interval in frequentist statistics. The most popular confidence level is 95%, however other levels, such 90% or 99%, are occasionally used for computing confidence intervals. A constant that indicates the intended degree of significance based on the normal distribution is multiplied by the standard error once it has been calculated to get the confidence interval. Intervals with 95% certainty have a constant value of 1.96.

The number of people in the sample that felt protecting the rights of those with unpopular views is a very important component of a strong democracy is 1200*.78 = 936

Yes, the sample size is large enough, since the expected number of successes is 936, and the expected number of failures is 1200-936= 264, both of which are greater than or equal to 10.

95% Confidence interval :    

At α = 0.05, two tailed critical value

Lower Bound =[tex]0.757[/tex]

Upper Bound = 0.803

(0.757, 0.803)

To know more about 95% Confidence interval visit:
https://brainly.com/question/15683202

#SPJ4


[tex]2 \frac{3}{5} \div 5\frac{3}{8} [/tex]
in its simplest form:?​

Answers

The expression 2 3/5 ÷ 5 3/8 given is written in simplest form as  104/215

How to write the expression in simplest form

The problem is a division problem in mixed numbers

converting to improper fractions we have

= 2 3/5 ÷ 5 3/8

= 13/5 ÷ 43/8

inverting the divisor to change to multiplication

= 13/5 ÷ 43/8

= 13/5 * 8/43

= 104/215

The division problem is simplest form give 104/215

Learn more about division at;

https://brainly.com/question/25289437

#SPJ1

the base of s is a circular disk with radius 5r. parallel cross-sections perpendicular to the base are squares.

Answers

The volume of the solid is 250r³ when  the base of s is a circular disk with radius 5r.

Given that,

We have to discover the described solid's volume, v. The parallel cross-sections perpendicular to the base of s are squares, and the base of s is a circular disk with radius 5r.

We know that,

A cylinder is a solid with square cross sections parallel to the bases and circular bases. Its height is equal to the base circle's diameter.

The radius = 5r,

So the height = diameter = 10r

Volume of the cone is base × height= πr²×h

=π(5r)²×(10r)

=250πr³

Therefore, The volume of the solid is 250r³ when  the base of s is a circular disk with radius 5r.

To learn more about volume visit: https://brainly.com/question/12027549

#SPJ4

let x be a continuous random variable. we know that it takes values between 0 and 3, but we do not know its distribution or its mean and variance. we are interested in estimating the mean of x, which we denote by h. we will use 1.5 as a conservative value (upper bound) for the standard deviation of x. to estimate h, we take n i.i.d. samples x1,x2,...,xn, which all have the same distribution as x, and compute the sample mean

Answers

The randomized variable's sample distribution would be as follows: H+/- 1.96 * √3)/√n

A random variable in mathematics would be a variable for whom the value is the numerical result of an unpredictable event and a random number generator X on a sample space. S denotes the rule that gives each of the outcomes S in a collection a numerical value.

In this case, we have assumed that x is a constant random variable with values ranging from 0 to 3, but we are unsure of its distribution, mean, or variance.

The mean of x, which we designate by the letter h, is what we're now interested in estimating. To provide a conservative estimate (upper bound), we'll choose 1.5 for the standard deviation of x.

In this case, we must estimate h. To do this, we select the sample size n, the samples x1, x2,.., xn, all of which share the same dissemination as x, and accurately measure the sample mean.

Therefore, we have used a normal distribution to determine the variation of x as (0,6),

And it equals to 3, then apply CLT resulting in

=> H+/- 1.96 * √3)/√n

To know more about Random variable here.

https://brainly.com/question/17238189

#SPJ4

help me solve this please

Answers

Thus, the required trig values corresponding to the cosФ = -3/5 have been shown,

What are trigonometric equations?

These are the equation that contains trigonometric operators such as sin, cos.. etc.. In algebraic operations.

Here,
cosФ = -3/5
cosФ = -3/5 =  base / hypotenuse,

perpendicular = √5² - 3² = 4

sinФ = perpendicular / hypotenus
       = 4/5

Simultaneously,
cosecФ = 5/4
secФ = -5/4
tanФ = -4/3
cotФ = -3/4

Here,
sin and cosec are positive because this function as a positive positive value between π/2 and π, while else trigonometric function as a negative value for π/2 to π.

Thus, the required trig values are shown above.

Learn more about trigonometry equations here:

brainly.com/question/22624805

#SPJ1

transformation(s) can map APQR onto ASTU?
Which
O translation only
O rotation only
O rotation, then reflection
O reflection, then translation

Answers

Answer:
D.) reflection, then translation

Explanation:
The triangles shown are congruent by the SSS congruent theorem.

Hopes this helps <3

Los Angeles workers have an average commute of 27 minutes. Suppose the LA commute time is normally distributed with a standard deviation of 15 minutes. Let X represent the commute time for a randomly selected LA worker. Round all answers to 4 decimal places where possible.
a. What is the distribution of X? X ~ N(,)
b. Find the probability that a randomly selected LA worker has a commute that is longer than 35 minutes.
c. Find the 70th percentile for the commute time of LA workers. minutes

Answers

(a) Distribution of X is a bell curve with mean of  27 min

(b) The probability of a randomly selected LA worker has a commute that is longer than 35 minutes is 79.81%

(c) 70th percentile for the commute time of LA workers is 34.866

Given,

Mean = 27 minutes

Standard Deviation = 15 minutes

a. The distribution is a bell curve, with the peak of the curve corresponding to the mean = 27 minutes.

b. We have to find the probability that a randomly selected LA worker will have a commute of over 35 minutes. This value will fall on the right hand side of the bell curve, i.e, on the right side of the mean. For that, we need to find the z-score for 35.

z score, z = [tex]\frac{35-27}{15} = 0.53[/tex]

Using the z-tables, we get the z-value to the left of 0.53 as 0.2019, and therefore, the z-value to the right will be 1 - 0.2019 = 0.7981, which gives us a probability percentage of around 79.81%.

c. Here, we have to find the 70th percentile for the commute time for the LA workers. Let us take the marker as 0.70 on the bell-curve, so a z-score that gives us a value of 0.70 should be the right answer.

The answer to that is 0.5244.

Now, a little calculation:

0.5244 x 15 = 7.866 = x - 27

x = 34.866

Final answers:

a. A bell curve

b. 0.7981

c. 34.866

To learn more about probability,

brainly.com/question/30034780

#SPJ4

a. The distribution is a bell curve, with the peak of the curve corresponding to the mean = 27 minutes.

b. Using the z-tables, we get the z-value to the left of 0.53 as 0.2019, and therefore, the z-value to the right will be 1 - 0.2019 = 0.7981, which gives us a probability percentage of around 79.81%.

c. Here, we have to find the 70th percentile for the commute time for the LA workers. Let us take the marker as 0.70 on the bell-curve, so a z-score that gives us a value of 0.70 should be the right answer.

The answer to that is 0.5244.

0.5244 x 15 = 7.866 = x - 27

x = 34.866

Messages arrive to a computer server according to a Poisson distributionwith a mean rate of 17 per hour. Determine the length of an interval oftime such that the probability that no messages arrive during this intervalis 0.94.

Answers

The length of an interval of time such that the probability that no messages arrive during this interval is 0.94 is 56.88 seconds.

We have to find the length of the time interval that the probability of no messages arrive during this interval is 0.94

Let X be the no.of messages arrive to the server

X follows the poisson distribution with mean λ = 17

Let t be the time interval

The probability of no messages arrive during this interval is 0.94

Then P(0) = 0.94

e⁻⁽λt⁾ = 0.94

λt = ln0.94

17 t   = ln(0.94)

t = ln(0.94)/17

t = 0.026872 / 17 hrs

t = 0.0158

Convert 0.0158 hours to seconds

That is t=0.0158(60)(60)

 =56.88seconds

Hence in 56.88 seconds the time interval that the probability of no messages arrive during this interval is 0.94 .

To learn more about probability

visit - https://brainly.com/question/13604758

#SPJ4

Add a department store a coat that normally sells for $50 it's on sale for $35 what is the percent of the decrease? HELP

Answers

Answer: 30%

Step-by-step explanation: To find the percent of decrease, you have to subtract the greater number from the smaller one, then divide it by the first number that comes in the sentence x 100. So, using that formula, we subtract 35 from 50 to get 15, then we do 15/50 x 100 to get 30 or 30%. I hope this helped you understand a little more about the percentage decrease.

let sn be the number of successes in n independent bernoulli trials, where the probability of success for each trial is 1/2. evaluate the following limits.
(a) n→[infinity]lim P( 2n− 3n ≤S n ≤ 2n​ + 3n)
(b)n→[infinity] lim P( 2n− 4n ≤S n ≤ 2n+ 4n)
(c) n→[infinity]lim P(2n−20≤Sn ≤ 2n+20)

Answers

The following limits -

Option a) = 1

Option b) = 0

Option c) = 0

According to the question given,

The probability of success of each trial is = [tex]\frac{1}{2}[/tex]

Let us consider options a),

n→[infinity]lim P( [tex]2n - 3n \leq S n \leq 2n + 3n[/tex] ),

We could conclude that if n tends to infinity, the probability of having [tex]2n - 3n[/tex] successes in n number of trials = the probability of having [tex]2n - 3n[/tex] successes in n number of trials.Since in the question, the probability of success for each trial is 1/2, so we could say that, [tex]2n - 3n[/tex] successes = [tex]2n - 3n[/tex] successes

Therefore, the limit is = 1

Taking option b),

We could conclude that if n tends to infinity, the probability of having [tex]2n - 4n[/tex] successes in n number of trials = 0Since in the question, the probability of success for each trial is 1/2, so we could say that, [tex]2n - 4n[/tex] successes = 0

Therefore, the limit is = 0

Taking option c),

We could conclude that if n tends to infinity, the probability of having [tex]2n - 20[/tex] successes in n number of trials = 0Since in the question, the probability of success for each trial is 1/2, so we could say that, [tex]2n - 20[/tex] successes = 0

Therefore, the limit = 0

Therefore, the following limits are - a) 1, b) 0, c) 0

To know more about Probability,

https://brainly.com/question/11234923

#SPJ4

Find the area of this triangle. Round to the nearest tenth. PLEASE HELP

Answers

Answer:

74.8 m²

Step-by-step explanation:

[tex]\boxed{\begin{minipage}{7.6 cm}\underline{Area of a triangle} \\\\$A=\dfrac{1}{2}ab \sin C$\\\\where:\\ \phantom{ww}$\bullet$ $C$ is the angle. \\ \phantom{ww}$\bullet$ $a$ and $b$ are the sides enclosing the angle. \\\end{minipage}}[/tex]

From inspection of the given triangle:

C = 115°a = 11 mb = 15 m

Substitute the values into the formula and solve for area:

[tex]\implies A=\dfrac{1}{2} \cdot 11 \cdot 15 \cdot \sin 115^{\circ}[/tex]

[tex]\implies A=\dfrac{165}{2} \sin 115^{\circ}[/tex]

[tex]\implies A=74.77039243[/tex]

[tex]\implies A=74.8\; \sf m^2\;(nearest\;tenth)[/tex]

Therefore, the area of the triangle is 74.8 m² rounded to the nearest tenth.

In order to understand the relation between smoking and hypertension (HT), a tea of researchers designed a study that looks at the numbers of smokers and nonsmokers with no HT, pre-HT, stage 1 HT, and stage 2 HT. Which one of the following statements is correct? a. A chi-square test of independence should be run for the study, because the outcome smoker vs. nonsmoker-is a continuous variable, and the grouping variable-no HT, pre-HT stage 1 HT, and stage 2 HT-categorizes the sample into four independent groups. b. A chi-square test of independence should be run for the study, because the outcome smoker vs. nonsmoker-is a dichotomous variable, and the grouping variable-no HT, pre-H stage 1 HT, and stage 2 HT-categorizes the sample into four independent groups. c. An independent samples t test should be run for the study, because the outcome-smoker vs. nonsmoker-is a categorical variable, and the grouping variable no HT, pre-HT, stage 1 HT, and stage 2 HT-categorizes the sample into four independent groups. d. An independent samples t test should be run for the study, because the outcome-smoker vs. nonsmoker-is a continuous variable, and the grouping variable no HT, pre-HT, stage 1 HT, and stage 2 HT-categorizes the sample into matched (or dependent) groups.

Answers

Correct option is C

In order to understand the relation between smoking and hypertension (HT) a tea of researchers designed a study that looks at the numbers of smokers and nonsmokers with no HT, pre-HT, stage 1 HT, and stage 2 HT.

An independent samples t test should be run for the study because the outcome smoker vs nonsmoker is a categorical variable and the grouping variable no HT, pre-HT, stage 1 HT, and stage 2 HT-categorizes the sample into four independent groups.

Variables

In Maths, a variable is an alphabet or term that represents an unknown number or unknown value or unknown quantity. The variables are specially used in the case of algebraic expression or algebra. For example, x+9=4 is a linear equation where x is a variable, where 9 and 4 are constants.

Probability

Probability is simply likely something is to happen. Whenever unsure about the outcome of an event, we can talk about the probabilities of certain outcome. The analysis of events governed by probability is called statistics.

Correct option is C

To learn more about Probability and Variable visit:

brainly.com/question/29069559

#SPJ4

Determine the slope of the line that contains the points with coordinates (1, 5) and (-2, 7).​

Answers

Answer:

2/3x

Step-by-step explanation:

the scientist creates an equation that models her data for each tree so that she can predict the diameter in the future. complete a linear equations that fits the data for tree 1, where x is the year and y is the trunk diameter, in inches. click on the variables and number you want to select and drag them into the boxes.

Answers

The linear equation obtained is y = 0.3X + 18.3

What is linear equation?

A linear equation is an equation that can be written in the form ax + b = 0, where a and b are real numbers and x represents an unknown variable. The solutions of linear equations are often graphed on a coordinate plane, forming a straight line. Linear equations can be used to model many real-world situations, such as population growth, the rate of change of a physical quantity, or the total cost of an item. Linear equations can also be used to solve systems of equations, where several equations must be solved simultaneously.

Given that data :
x = year ;
y = trunk diameter, in inches
Year ________trunk diameter
1 __ 18.6
3 __ 19.2

5 __ 19.8
7 __ 20.4
9 ___21.0
11 __ 21.6
13 __ 22.2
Using the linear regression calculator :
The linear equation obtained is :
y = 0.3X + 18.3
Where ;
Slope = 0.3 ; intercept, c = 18.3

To learn more about linear equation
https://brainly.com/question/2030026
#SPJ4

What are the vertex and range of y = |2x + 6| + 2? (0, 2); 2 < y < ∞ (0, 2); −∞ ≤ y < ∞ (−3, 2); 2 < y < ∞ (−3, 2); −∞ ≤ y < ∞

Answers

The vertex and the range of y = |2x + 6| + 2 are (c) (-3, 2); 2 ≤ y < ∞

How to determine the vertex and the range?

From the question, we have the following parameters that can be used in our computation:

y = |2x + 6| + 2

The above equation is an absolute value function

An absolute value function represented as

y = a|x - h| + k

Where

Vertex = (h, k)

So, we have

y = |2x + 6| + 2

This gives

y = 2|x + 3| + 2

This means that the vertex is

Vertex = (-3, 2)

Remove the x value

y = 2

Because the leading coefficient is positive, then the vertex is a minimum

i.e. y ≥ 2

So, the range is 2 ≤ y < ∞

Read more about vertex at

brainly.com/question/1480401

#SPJ1

Solve the equation. *Will give Branliest!*

-1/6x-5=2/3x

Answers

Answer:

[tex]x=-6[/tex]

Step-by-step explanation:

Solve for x.

Combine [tex]x[/tex] and [tex]\frac{1}{6}[/tex].

Combine [tex]x[/tex] and [tex]\frac{2}{3}[/tex].

[tex]-\frac{x}{6} -5=\frac{2x}{3}[/tex]

Move all terms containing x to the left side of the equation.

Subtract [tex]\frac{2x}{3}[/tex] from both sides of the equation.

[tex]-\frac{x}{6} -5-\frac{2x}{3}=0[/tex]

To write [tex]-\frac{2x}{3}[/tex] as a fraction with a common denominator, multiply by [tex]\frac{2}{2}[/tex].

[tex]-\frac{x}{6}-\frac{2x}{3}*\frac{2}{2}-5 =0[/tex]

Write each expression with a common denominator of 6, by multiplying each by an appropriate factor of 1.

[tex]-\frac{x}{6}-\frac{2x*2}{6}-5 =0[/tex]

Combine the numerators over the common denominator.

[tex]\frac{-x-2x*2}{6}-5 =0[/tex]

Simplify the numerator.

Factor [tex]x[/tex] out of [tex]-x-2x*2[/tex].

[tex]\frac{x(-1-2*2)}{6}-5 =0[/tex]

[tex]\frac{x(-1-4)}{6}-5 =0[/tex]

[tex]\frac{x*-5}{6}-5 =0[/tex]

Move the negative in front of the fraction.

[tex]-\frac{5x}{6} -5=0[/tex]

Add 5 to both sides of the equation.

[tex]-\frac{5x}{6}=5[/tex]

Multiply both sides of the equation by [tex]-\frac{6}{5}[/tex].

[tex]-\frac{6}{5}(-\frac{5x}{6})=-\frac{6}{5}*5[/tex]

Simplify the left side by cancelling the common factors of 5 and 6.

[tex]x=-\frac{6}{5} *5[/tex]

Move the leading negative in [tex]-\frac{6}{5}[/tex] into the numerator.

[tex]x=\frac{-6}{5} *5[/tex]

Cancel the common factor of 5.

[tex]x=-6[/tex]

A rating/review would be much appreciated. Wish you a happy holiday season!

Answer:

x = - 6

---------------------------------

Given equation:

-1/6x - 5 = 2/3x

Fist, multiply both sides by 6 to clear the fraction:

6(-1/6)x - 6(5) = 6(2/3)x- x - 30 = 4x

Collect the terms with the variable and solve for x:

4x + x = - 305x = - 30x = - 30/5x = - 6

Please help me!!!!!!!!!!!!

Answers

Answer:

A

Step-by-step explanation:

change 'its' to   it's    which is a contraction for 'it is'

form differential equations for all circles in the xy plane

Answers

The differential equation that can represent all circles in the xy plane is given as follows:

[tex]\frac{dy}{dx} = -\frac{x - h}{\sqrt{r^2 - (x - h)^2}}[/tex]

How to obtain the differential equations for a circle?

The equation of a circle of center (h,k) and radius r is given as follows:

(x - h)² + (y - k)² = r².

The equation can be arranged to isolate the variable y as follows:

(y - k)² = r² - (x - h)²

y - k = square root (r² - (x - h)²)

y = square root (r² - (x - h)²) + k

To obtain the differential equation, then this equation must be differentiated towards variable x as follows:

[tex]\frac{dy}{dx} = \frac{d}{dx}(\sqrt{r^2 - (x - h)^2} + k)[/tex]

Applying the chain rule, along with the derivative for the square root, we have that:

[tex]\frac{d}{dx}(\sqrt{r^2 - (x - h)^2} + k) = -\frac{x - h}{\sqrt{r^2 - (x - h)^2}}[/tex]

Meaning that the differential equation is given as follows:

[tex]\frac{dy}{dx} = -\frac{x - h}{\sqrt{r^2 - (x - h)^2}}[/tex]

More can be learned about the equation of a circle at https://brainly.com/question/1506955

#SPJ1

Could I have some help with the question below:
In each part, find the two unit vectors in R^2 that satisfy the given conditions:
The two unit vectors parallel to the line y=5x+5 are: _ and _
The two unit vectors parallel to the line 2x+4y=1 are: _ and _
The two unit vectors perpendicular to the line y=2-5x are: _ and _

Answers

The two unit vectors parallel to the line y=5x+5 are [tex]u_{+}[/tex] =⟨1, 5⟩/√26 and [tex]u_{-}[/tex] =⟨1, 5⟩/√26.

What are vectors?

Vectors, in Maths, are objects which have both, magnitude and direction. Magnitude defines the size of the vector. It is represented by a line with an arrow, where the length of the line is the magnitude of the vector and the arrow shows the direction.

Part A:

The given equation of line is y=5x+5.

It's all about slope. The vectors parallel to the line must have the same slope as the line. The line here is y=5x+5., and has slope 5. We can say the rise is 5 and the run is 1 and write the vector.

[tex]u_{\pm}[/tex] =⟨1, 5⟩

Magnitude =|u| =√(1²+5²)

= |±√26|

= √26

So, the two unit vectors are

[tex]u_{+}[/tex] =⟨1, 5⟩/√26 and [tex]u_{-}[/tex] =⟨1, 5⟩/√26

Part B: 2x+4y=1

x+2y=1/2

y=-x/2+1/4

Here slope is -1/2, where rise is -1 and run is 2

[tex]u_{+}[/tex]= ⟨2, -1⟩ and [tex]u_{-}[/tex]= ⟨-2, 1⟩

Magnitude =|u| =√(2²+(-1)²)

= √5

So, the two unit vectors are

[tex]u_{+}[/tex] =⟨2, -1⟩/√5 and [tex]u_{-}[/tex] =⟨-2, 1⟩/√5

Part C: The given equation is y=2-5x

Here slope is -5, slope perpendicular to line is -1/5

So, rise is -1 and run is 5

[tex]u_{\pm}[/tex] =⟨5, -1⟩

Magnitude =|u| =√(5²+(-1)²)

= √26

So, the two unit vectors are

[tex]u_{+}[/tex] =⟨5, -1⟩/√26 and [tex]u_{-}[/tex] =⟨-5, 1⟩/√26

Therefore, the two unit vectors parallel to the line y=5x+5 are [tex]u_{+}[/tex] =⟨1, 5⟩/√26 and [tex]u_{-}[/tex] =⟨1, 5⟩/√26.

Learn more about the vectors here:

https://brainly.com/question/29740341

#SPJ2

Given m = 2 and the point (6, 8), what is the point-slope form of the equation?​

Answers

Answer:

y - 8 = 2 (x - 6)

Step-by-step explanation:

The point-slope equation is ...

y - y1  = m ( x - x1 )

In the coordinate (6,8)

x = 6 and y = 8

m = 2

Plug in values to find the point-slope form

y - 8 = 2 (x - 6)

We are done!

Calculate the difference and enter below. 3-9

Answers

Answer:

-6 is your answer

Step-by-step explanation:

I hope that's right

based on his past record, luke, an archer for a college archery team, has a probability of 0.90 of hitting the inner ring of the target with a shot of the arrow. assume that in one practice luke will attempt 5 shots of the arrow and that each shot is independent from the others. let the random variable x represent the number of times he hits the inner ring of the target in 5 attempts. the probability distribution of x is given in the table. P(X) 0 000001 0,00045 0.00810 0.07290 03280S 0.59049
What is the probability that the number of times Luke will hit the inner ring of the target out of the 5 attempts is less than the mean of X?
(A) 0.40951
(B) 0.50000
(C) 0.59049
(D) 0.91854
(E) 0.99144

Answers

The probability that Luke will touch the target's inner ring fewer times than the mean of X out of five attempts is 0.40951.

What is probability ?

Probability refers to possibility.

A random event's occurrence is the subject of this area of mathematics. The range of the value is 0 to 1. Mathematics has incorporated probability to forecast the likelihood of various events.

The degree to which something is likely to happen is basically what probability means.

You will understand the potential outcomes for a random experiment using this fundamental theory of probability, which is also applied to the probability distribution.

Knowing the total number of outcomes is necessary before we can calculate the likelihood that a specific event will occur.

According to our question-

P(target) equals 0.90

Let the random variable X stand in for the number of times the arrow is fired at the target's ring.

The probability distribution of X is

  x:         0              1                2               3               4               5

P(x):    0.00001   0.00045  0.00810   0.07290   0.32805   0.59049

Hence, The probability that Luke will touch the target's inner ring fewer times than the mean of X out of five attempts is 0.40951.

learn more about probability click here:

https://brainly.com/question/13604758

#SPJ4

What do I have to fill out in the blanks?

Answers

Opposite sides of a parallelogram are equal.

Opposite sides of a parallelogram are equal in the second case

RT is congruent to RT is obvious.

ΔRST and ΔRUT are congruent because a diagonal of a parallelogram divides it into two congruent triangles.

We know pair of interior alternate angles are equal.

What is congruency?

We know two similar planer figures are congruent when we have sides or angles or both that are the same as the corresponding sides or angles or both.

Given, Line segment TU is congruent to line segment RS because in a parallelogram opposite are equal same reason for line segment RU and ST.

We also know that the diagonal of a parallelogram divides the parallelogram into two equal triangles hence ΔRST and ΔRUT are congruent.

We also know that the pair of alternating angles are equal hence,

∠RST and ∠TRU are congruent.

learn more about congruency here :

https://brainly.com/question/7888063

#SPJ1

What is 19x( _________X26)=(19X39)X26

Answers

Answer:

(39 * 26)

Step-by-step explanation:

If (19 * 39) * 26

19 * (39 * 26)

Your answer is 39.

indicate if dijkstra's algorithm will be able to find the shortest path for this particular graph. (assume the source vertex is m) yes no it will loop forever the algorithm will never start

Answers

By using Dijkstra's algorithm, it can be concluded that-

It will loop forever.

Third option is correct

What is Dijkstra's algorithm?

Dijkstra's algorithm is used to find the shortest path between the nodes of a graph.

Here, a node is fixed which is called the source node and the distance from the source node to all other nodes are calculated.

The minimum distance gives the shortest path.

For the first option,

The first option is incorrect because the shortest path will not be obtained. There does not exist path between every pair of vertices

For the second option

The second option is incorrect because all the conditions of Dijkstra's algorithm are fulfilled but here, infinite looping takes place.

For the fourth option

The fourth option is incorrect because there exist a path between the point M and every other vertex in the graph.

So the third option is correct

To learn more about Dijkstra's algorithm, refer to the link-

https://brainly.com/question/15392537

#SPJ4

Complete Question

The complete question has been attached

A forest covers 41000acres. A survey finds that 0.2% of the forest is old-growth trees. How many acres of old-growth
trees are there?

Answers

The answer is 82acres! All you need to do is find out what 0.2% of 41000 is. An easy way to do that is to multiply the percentage by the number. For example, 10% of 592 would be 0.10x592=59.2 and 46% of 259 would be 0.46x259=119.14. So 0.2% of 41000 is 0.002x41000=82. I hope this helps!

A man distributes his savings of 4000 among his three sons in the ratio 4:3:3. Find the amount received by his first son.

Answers

Answer:

First son: 4

Second son: 3

Third son: 3

Sum ratio =10

First son= 4/10*4000=1600

Triangle Congruence: Question 3
In the proof, what is the reason for (5) ?
Given: V W║Z Y
W X=Y X
Prove: ΔVWX ΔXYZ
Select one:
SSA
H
SAS
ASA
SSS

Answers

In the proof the reason for (5)ΔVWX ≅ ΔXYZ  is  ASA , the correct option is (d) .

In the question ,

it is given that ,

Given: VW parallel to ZY  and WX = YX

to Prove: ΔVWX ≅ ΔXYZ .

the statement to prove ΔVWX ≅ ΔXYZ are given as

(1) VW parallel to ZY      ....given

(2) m∠W = m∠Y    ...alternate interior angle .

(3) WX = YX     ...given

(4) m∠VXW = m∠ZXY      .....vertically opposite angle

(5) ΔVWX ≅ ΔXYZ

In statement (5) , Since two angles and 1 side is included in proving the given two triangles congruent ,

the reason will be ASA congruence , that is option (d) .

Therefore , the reason for (5) is ASA .

Learn more about Congruence here

https://brainly.com/question/27292864

#SPJ4

Other Questions
what is one of the most sought-after skills employers screen for when hiring public relations talent? the american torture policys extent was not fully uncovered until the release of a senate committee report in 2014. this report showed that torture was used systematically at secret u.s. prisons around the world. the report also concluded, however, that an incredible amount of information was obtained from prisoners through the use of these brutal methods. State True or False your answer: a. Trueb. False glycogen synthesis requires a protein primer (glycogenin) and an activated glucose precursor (udp-glucose). which anatomic abnormalities are found in tetralogy of fallot overriding aorta aortic stenosis Explain the importance of paying cash for college. then, using examples from the text, share ideas of how to cash flow your postsecondary education. 1. occasionally, a layer of cool air forms under a layer of warmer air in the troposphere. this change from the normal temperature profile in the troposphere is called a L Addressed in Articlethe11. Led by theIII. The president must beyears oldIV. The president must becitizen.V. The president is the commander in chief ofVI. The president serves a term that lastsyears.VII. The president can nominatejustices.VIII. The president can reject billproposals - this is called aIX. The president can makewith foreign countriesX. Despite popular opinion,the president cannot declareXI. At the state level, theleader of this branch iscalled the-bornI. There areL Addressed in ArticleThis branch of govemment is led by theat the national levelmembers on theL. Addressed in ArticleII. Led by theIII. Congress is bicameral, which meansit hassides.IV. Members in the House of Representativesmust beyears oldV. Members in the House of Representativesserve foryear terms.VI. Members in the Senate must beyears old.VIL Members in the Senate serve foryear termsVill. Both sides of Congress play a role in impeaching thewhich means to accuse of aX. There areTHE UNITEDSTATESGOVERNMENTSenators for each state, whichtotalmeans there areX. There aremembers in theHouse of Representatives. Thismeans there aretotalmembers in CongressXI. People often think that thepresident can declareis a power given toCongressVL The Supreme Court determinesis guilty of a crime i capacity requirements are computed by multiplying the number of units scheduled for production at a work center by the: group of answer choices the unit resource requirements and then adding in the setup time. the setup time and then subtracting the unit resource requirements. the setup time and then adding in the unit resource requirements. the unit resource requirements and then subtracting the setup time. Answer these three questions for full points. Solve the system of inequalities. -7x+3y 156x-5y For the same samples statistics, which level of confidence would produce the widest confidence interval? Explain your reasoning. Choose the correct answer below.a. 99%, because as the level of confidence increases, Zc increases.b. 99%, because as the level of confidence increases, Zc decreases.c.90%, because as the level of confidence decreases, Zc increases.d. 90%, because as the level of confidence decreases, Zc decreases. Find the measure of QTS.Image posted !! Please help giving brainliest for whoever gets it right a compound forms a dense yellow precipitate when treated with iodine and sodium hydroxide. the compound must be: Complete the sentences with look or be. 1. He... generous. He always helps poor families. 2. She... cold and unfriendly. In fact, she is really kind. 3. My sisters...sociable. Actually, they are quiet. 4. My uncle... hardworking. He grows many vegetables in their yard. 5. Most of us think she...miserable today but teacher says he looks cheerful. In each case below find an equation for the line with the given information about the linear equation and give the slope of each line. a) Vertical line through point (2, -65) b) Horizontal line through point (-2, 4) the f-test for anova in a regression model with 4 predictors and 47 observations would have how many degrees of freedom? what is the f-statistic for the f-test for joint significance for the variables that are omitted between regression 1 and regression 2? how did industrialization change how goods were produced? consider the factory system, and specialization of labor. You have sampled a population in which you know that the percentage of the homozygous recessive genotype (aa) is 36%. Using that 36%, calculate the following:a. The frequency of the "aa" genotype. Answer: 36%, as given in the problem itself.b. The frequency of the "a" allele. Answer: The frequency of aa is 36%, which means that q2 = 0.36, by definition. If q2 = 0.36, then q = 0.6, again by definition. Since q equals the frequency of the a allele, then the frequency is 60%.c. The frequency of the "A" allele. Answer: Since q = 0.6, and p + q = 1, then p = 0.4; the frequency of A is by definition equal to p, so the answer is 40%.d. The frequencies of the genotypes "AA" and "Aa." Answer: The frequency of AA is equal to p2, and the frequency of Aa is equal to 2pq. So, using the information above, the frequency of AA is 16% (i.e. p2 is 0.4 x 0.4 = 0.16) and Aa is 48% (2pq = 2 x 0.4 x 0.6 = 0.48).e. The frequencies of the two possible phenotypes if "A" is completely dominant over "a." Answers: Because "A" is totally dominate over "a", the dominant phenotype will show if either the homozygous "AA" or heterozygous "Aa" genotypes occur. The recessive phenotype is controlled by the homozygous aa genotype. Therefore, the frequency of the dominant phenotype equals the sum of the frequencies of AA and Aa, and the recessive phenotype is simply the frequency of aa. Therefore, the dominant frequency is 64% and, in the first part of this question above, you have already shown that the recessive frequency is 36%. roblyer per the course syllabus, for what exceptional situations will i consider changing your final course grade? group of answer choices